Add last few weeks of Phys 102 solutions.
[course.git] / latex / problems / Serway_and_Jewett_8 / problem27.29.tex
1 \begin{problem*}{27.29}
2 A certain waffle iron is rated at $1.00\U{kW}$ when connected to a
3 $120\U{V}$ source.  \Part{a} What current does the waffle iron
4 carry?  \Part{b} What is its resistance?
5 \end{problem*}
6
7 \begin{solution}
8 \Part{a}
9 Using the power formula
10 \begin{align}
11   P &= IV \\
12   I &= \frac{P}{V} = \ans{8.33\U{A}}
13 \end{align}
14
15 \Part{b}
16 Using Ohm's law
17 \begin{align}
18   V &= IR \\
19   R &= \frac{V}{I} = \frac{V^2}{P} = \ans{14.4\U{\Ohm}}
20 \end{align}
21 \end{solution}